Skip to Main Content

PrepTest 79, Game 2, Question 7

Transcript

If O is the sole ranger assigned to area 2 then which one of the following could be the complete assignment of rangers to area 3? So we can go ahead and drop O into the diagram and then close off these other 2 spaces here in area 2. Now if O is the only variable that can go 2, that leaves us five variables that need to be placed.

So we have JKLM and P that need to placed. Now, we already know that M has a permanent home. So we're still gonna need to place JKL and P. And none of those variables can go in the second spot. So this is gonna create sort of a spatial relationship issue here, where we're gonna need to be aware of how many spaces that we have versus the variables that we need.

Now if we just look at O and 2 and we look at the remaining spaces that we have left, we're going to need to have sort of a two, three arrangement here. So three of the variables are gonna need to go into one of the areas, and two of the variables are gonna need to go into the other areas. In order to meet the requirements of one of our rules that we have every variable assigned to an area.

So we need to account for the fact that we've lost two spaces in area 2 and then figure out the distribution to area 1 and 3. So again, one's going to have two variables, the other one is going to have three variables. So if we tried to put four of those remaining variables in one of these spaces, it's not gonna work, remember?

Because we have to have, we can only have a maximum of three variables per area. So that's how we sort of get our two, three distribution there. So the correct answer is C, M and P and could be a complete listing of the variables assigned to area 3. So if we just drop M, we already have M there, but if we drop P into 3. We want to then take through the rest of our roles and make sure we're meeting those requirements.

So because O isn't 2, we know that J and K must go in the same area. So we have J and K going in the area 1 and then we also meet our Requirement here where one of LM or LK, so we have LK. Everything looks good here, so we have our three variables in area 1 and two variables in area 2, which was the distribution of those variables that we needed, so this is our correct answer.

But let's look at some of the other answer choices in this section, because they were a bit tricky and really did count on your understanding the distribution of the remaining variables. So answer choice A, having just M in area 3 is not okay. So remember that we have those five variables remaining. After we place O in 2, M automatically already goes in 3, but we need to have a three, two distribution between the remaining areas.

Meaning one area gets three, another area gets two. So we cannot put a single variable in one of the areas or we're gonna overload the other area and then violate a rule. So answer choice B, we have L and M going in 3, so if we put L down here in 3 we're gonna be left with J, K, and P. So we're gonna need to make sure that we have K going here, since L is already gonna be down here with 3 and we can only have L or M or L or K.

So then we would need to have L and K together. But remember that since O is in 2, K is bringing along a friend, so J would go here as well. But, we then won't be able to place P, so one, P can't go in the first area. And then 2, the area is going to be full. So there's a lot of problems with answer choice B.

Answer choice D, we have J, K, and M going in area 3,so that's gonna leave L and P left over. P can't go in 1, and we're gonna end up with, so we have J and M, or K and M, in the same area, which means that L cannot go in area 3 since it can't go with both. But since K and M are together, that means that L is not going to be with either of them, so K and M can't be in the same area together.

And then answer choice E, we have J, M, N, P going in the third area. Well, we can just sort of toss this one out immediately, since we already know that we have O in 2. And that a complete and accurate list of where J is will also include K, so this is incorrect. So again, this answer, or this question, really focused on your understanding of sort of the distribution of these variables given the fact that we closed off area 2 after placing O there.

Read full transcript